LSAT and Law School Admissions Forum

Get expert LSAT preparation and law school admissions advice from PowerScore Test Preparation.

User avatar
 Dave Killoran
PowerScore Staff
  • PowerScore Staff
  • Posts: 5853
  • Joined: Mar 25, 2011
|
#41312
Complete Question Explanation
(The complete setup for this game can be found here: lsat/viewtopic.php?t=15710)

The correct answer choice is C)

From our Not Laws we know that M cannot be added first, and thus answer choice (C) is correct.
 Michaeljstrick3
  • Posts: 1
  • Joined: Mar 25, 2021
|
#91428
I see how answer choice B works, but I need help understanding how answer choice B is incorrect.

If the 2nd conditional rule states: Z1 --> (L--O), wouldn't the contrapositive be: (O--L) ---> (Z not 1).

Using this logic, the Lentils (L) couldn't be added first if Z is first, AND it also couldn't be added first if Z isn't first because it would have to be behind the Onions (O).

Could you please explain how my logic is off here?
 Robert Carroll
PowerScore Staff
  • PowerScore Staff
  • Posts: 1787
  • Joined: Dec 06, 2013
|
#91438
Michael,

It appears that you used a Mistaken Reversal of the contrapositive. The NECESSARY condition in that is that Z is not 1. That's not the sufficient condition of anything. So if Z is not 1...nothing happens. To think otherwise is a Mistaken Reversal.

Robert Carroll

Get the most out of your LSAT Prep Plus subscription.

Analyze and track your performance with our Testing and Analytics Package.